Maggie bought a pound of bananas and 4 boxes of cereal for $10.11. If each box of cereal cost $2.40, what was the price of a pound of bananas?
$0.51
$0.61
$1.51
$2.91

Answers

Answer 1

Answer:

$0.51

Step-by-step explanation:

If each box of cereal is $2.40, $2.40 multiplied by 4 is $9.60. $10.11 is the total, so $10.11 - $9.60 = $0.51.

Answer 2

Answer:

$0.51

Step-by-step explanation:

the person above me was correct


Related Questions

If You Know The Answer Help :)

Answers

Answer:

0.3

Step-by-step explanation:

Answer:

3/20

Step-by-step explanation:

1/5=4/20

7/20-4/20=3/20

Hope this helps!

im kinda dumb here ... need help again

Answers

Answer:

with what lol

Step-by-step explanation:

Answer:need me points

Step-by-step explanation:

Use the figure at the right. If JK = 2x + 12 and NO = 14, what is the value of x?

Answers

Answer:

X=8

Step-by-step explanation:

JK would equal 28, since it basically is double the side of that triangle. So now we have the equation, 2x+12=28. Now we solve using inverse operation. 28-12=16 and then 16 divided by 2 is 8, so x=8.

The required simplified value of x is 8, for the given triangle.

What is the midpoint?

A midpoint is a point on the line whose distance from both ends of the line is equal.

What is the triangle?

The triangle is a geometric shape that includes 3 sides and the sum of the interior angle should not be greater than 180°

Here,
The property of a triangle states that the line joining the midpoints of the two adjacent sides is equal to half of the third side.
So,
JK = 2NO
2x + 12 = 24
x = 16 / 2
x  = 8

Thus, the required simplified value of x is 8, for the given triangle.

Learn more about triangles here:

https://brainly.com/question/2773823

#SPJ2

Can you answer both and tell me the answer please

Answers

Answer:

7. 10^2

8. 3x3x3x3x3x3x3x3x3 = 19,683mi^2

The answer to 7 is 10^2 and 100.
10^2 is written with 10 and a small two toward the top right.
The answer to 8 is 3•3•3•3•3•3•3•3 and 19,683.

Seth sells candles for $14.35 each. Last
week, he sold 21 candles over 4 days. Seth
pays $2.40 for materials for every 3 candles.
How much did Seth earn last week?

Answers

Answer:284.55

Step-by-step explanation:divide 21 by 3=7  7times 2.40=16.8           14.35 times 21=301.55

if a plane travels 1,200 miles in 2 hours how many mph is the plane going

Answers

Answer:

600

Step-by-step explanation:

Sense 2 hours has passed you divide the amount of distance the plane has traveled by the amount of time it took. :)

Answer:

600 mph

Step-by-step explanation:

its the speed of the plane

the solution is distance travel which is 1,200 miles divided by the time 2 hours will be equal to 600 miles per hour (mph).

How do you do these questions?

Answers

Step-by-step explanation:

(a) Fₙ₋₁ > 0, so Fₙ₊₁ > Fₙ.  Each term is bigger than the one before it, so the function is increasing, meaning the series will diverge to infinity.

(b) Fₙ₊₁ / Fₙ = (Fₙ + Fₙ₋₁) / Fₙ

Divide.

Fₙ₊₁ / Fₙ = 1 + (Fₙ₋₁ / Fₙ)

Rewrite the second fraction using negative exponent.

Fₙ₊₁ / Fₙ = 1 + (Fₙ / Fₙ₋₁)⁻¹

Take the limit of both sides as n approaches infinity.

lim(n→∞) Fₙ₊₁ / Fₙ = 1 + lim(n→∞) (Fₙ / Fₙ₋₁)⁻¹

Substitute with φ.

φ = 1 + φ⁻¹

Solve.

φ² = φ + 1

φ² − φ − 1 = 0

φ = [ -(-1) ± √((-1)² − 4(1)(-1)) ] / 2(1)

φ = (1 ± √5) / 2

Since the ratio can't be negative:

φ = (1 + √5) / 2

What is the slope of the line y=-4

Answers

Answer:

slope is 0

Step-by-step explanation:

the equation describes a horizontal line with a y-intercept of -4. as the line is horizontal, the slope is 0

A local farm has a 0.5 acre field that was planted with 26,100 corn plants. One acre is equivalent to 43,500 square feet. Assuming the same planting density, how many corn plants could they plant on a field with an acre of 80,000 square feet

Answers

Answer:

96000 corn

Step-by-step explanation:

80,000/43500 = 1.84

1.84/0.5 = 3.678

3.678 x 26100 = 96000

what is 8455 divided by 89?

Answers

Answer:

95

Step-by-step explanation:

Answer:

95

Step-by-step explanation:

8455 divided by 89

Solve using the standard algorithm the area model the distributive property or the partial products method 2,809x4

Answers

Answer:

11,236

Step-by-step explanation:

i think this might be the answer but if it not alseat I try helping


Write a number that has a 3 that is 10
times the value of the 3 in 7.31
Your answer

Answers

Answer:

3

Step-by-step explanation:

7.31

3 in the tenths place

3 = 3/10 or 0.3

0.3*10=3

answer : 3
explain: 7.31
3 is in 10 place so
3= 3/10 or 0.3 ;)

2x+ 8y = -3
3x+ 6y = -4
Choose all answers that apply

Answers

multiply the top equation by 3, multiply the bottom equation by 4, then subtract the bottom equation from the top equation

multiply the top equation by 3, multiply the bottom equation by -2, then add the equations

answer the following...

Answers

It’s asking for which of the following will not prove that “l” is parallel to “n”... With this information we can choose an equation that has nothing to do with one of the line... giving us...
A) Angle one plus Angle four is equal to 180

Hope this helped!

Help Please I Really Need Help

Answers

Answer:

X=5

Step-by-step explanation:

3x+4=19

3x+4-4=19-4

3x=15

15/3x=

X=5

Hope this helps!

Use slope formula, m= y2, - y1 over X2-X1 to find the slope of a line that passes through the points (-3, 8) and (4.-6).

m=

use the slope-intercept form, y=mx+b to find the y-intercept (b) of the line

b=

what is the new equation written in slope-intercept form y=mx+b​

Answers

Answer:

m= -2

b= 2

y= -2x + 2

Step-by-step explanation:

Just did it

The slope of a line is m = -2 that passes through the points (-3, 8) and (4.-6), the y-intercept (b) of the line is 2, and the equation is y = -2x + 2.

What is the slope of the line?

The slope of a line is defined as the gradient of the line. It is denoted by m

Slope m = (y₂ - y₁)/(x₂ -x₁ )

Given that the line that passes through the points (-3, 8) and (4.-6).

Let

x₁ = -3, y₁ = 8

x₂ = 4, y₂ = -6

∵ Slope m = (y₂ - y₁)/(x₂ -x₁  )

Substitute values in the formula

m = (-6 - 8)/(4 -(-3))

m = (-14)/7

m = -2

y - 8 = m(x - (-3))

y - 8 = -2(x + 3)

y = -2x -6 + 8

y = -2x + 2

Since the slope-intercept form, y = mx+b

So the y-intercept (b) of the line,

b = 2

Thus, the slope of a line is m = -2 that passes through the points (-3, 8) and (4.-6), the y-intercept (b) of the line is 2, and the equation is y = -2x + 2.

Learn more about the Slope of the Line here:

brainly.com/question/14511992

#SPJ2

1.A cook has 2 3/4 pounds of ground beef.How many quarter-pound burgers can he make?

2.A child needs to take 1/6 tablespoons of medicine per day in 4 equal doses.How much medicine is in each dose?

3.How many 5/6 cup salt shakers can be filled from 24 cups of salt?

Answers

1- 11 quarter pounds burgers
2- 1/24 is a dose
3- 20 salt shakers

14. Solve and check the solution .2x + 1/3 =
4/3*
O X = 1/3
O X = 1/2​

Answers

[tex]2x + \frac{1}{3} = \frac{4}{3} [/tex]

[tex]2x = \frac{4}{3} - \frac{1}{3} [/tex]

[tex]2x = \frac{3}{3} [/tex]

[tex]2x = 1[/tex]

[tex]\color{hotpink}x = \frac{1}{2} [/tex]

Therefore , the correct option is :-

[tex]\color{yellow}\mapsto\color{teal}x =\color{magenta} \frac{1}{2} [/tex]

What is the value of 4x – 3y if x = 3 and y = 6?

Answers

Answer:

-6

Step-by-step explanation:

If x=3 and y=6, 4x-3y=-6 because 4x with x=3 makes the first number 12. 3y with y=6 makes the result 18, so now the equation is 12-18. 12-18=-6, so 4x-3y with x=3 and y=6 will equal -6.

Hopefully the answer and explanation helps!

Answer:

-6

Step-by-step explanation:

All you have to do is plug in the numbers. So it should look like 4(3) - 3(6). Now all you have to do is solve. 4 x 3 = 12. 3 x 6 = 18. Now subtract. 12 - 18 = -6

Evaluate the following expression: 24^3+7.2-15^5+6.2

Answers

Answer:

=−745,537.6

Step-by-step explanation:

Hope that helps

what is the slope of the line

Answers

Answer:

M=[tex]-\frac{3}{4}[/tex]

Step-by-step explanation:

M=[tex]\frac{rise}{run}[/tex]

From left to right, the line goes down. Therefore, it is a negative slope.

[tex]-\frac{3}{4}[/tex]

What is the value of 4x – 3y if x = 3 and y = 6?

Answers

4x — 3y if x = 3 and y = 6
I think 8 m

Answer:

The value is -6.

Step-by-step explanation:

First, you use subsitution.

4(3) - 3(6)

Then, you multiply.

12 - 18

Do the subtraction and you're left with -6 as your answer.

find each sum.
1)-9+(-2)
2)3.2+1.4
3)5.1+(0.7)
4)-2.2+(-3.8)

Answers

Answer:

1. -11

2. 4.6

3. 5.8

4. -6

Step-by-step explanation:

The answer:

1) - 11

2) 4.6

3) 5.8

4) 6.0

I hope this helps!!

Which of the ratio below is equivalent to 4:1? Select all that apply. A) 12:3 B) 24:6 C) 8:4 D)16:4 E) 10:6

Answers

Answer:

that would be a

Step-by-step explanation:

just took the test

Answer:

A, B,  D

Step-by-step explanation:

A bird goes from a higher location to a lower location at a
rate of 2 meters per second,
Which equation models the change in the bird's position at 30 seconde?
A. (-30)(-2) = -60
B. 30(-2) = -60
C. 30.2 = 60
D. 30(-2) = 60

Answers

I think it’s B not sure

Answer: It's B.

Step-by-step explanation: I took the test.

There are eight wires which need to be attached to a circuit board. A robotic device will attach the wires. The wires can be attached in any order, and the production manager wishes to determine which order would be fastest for the robot to use. Use the multiplication rule of counting to determine the number of possible sequences of assembly that must be tested.

Answers

Answer: 40320 sequence to test.

Step-by-step explanation:

we have given the following data:

No of wires which need to be attached to a circuit board = 8

What We need to find is the number of possible sequences of assembly that must be tested.

Using our multiplication rule of counting we arrive at,

Number of possible sequences of assembly is given by

8! = 8 x 7 x 6 x 5 x 4 x 3 x 2 x 1

= 40320

i.e. there are 8 choices for the first wire, 7 choices for the second wire, 6 choices for the third wire, 5 choice for the fourth wire same till the 8th wire.

Hence, there are 40320 possible sequences of assembly.

The graph of h(x) = x2 – 3 is a translation of the
graph of f(x)
by
units.

Answers

Answer: down, by 3 units

Step-by-step explanation: -3 is negative so you go down on the graph 3 times

Answer:

down by 3 units

Step-by-step explanation:

i just did this assignment on edg

In Earnest's yard, the grass grew 2 inches. The next week, Earnest cut 2 inches off of the grass. The total change was 0 inches. Which of the following situations
also has a total change of 0?
I put 5 pounds of birdseed in the feeder. Then, birds ate 5 pounds of birdseed.
It rained 2 inches on Tuesday. Then, it rained 2 inches on Wednesday.
It Snowed 4 inches last week. Then, 8 inches of snow melted during this week.
A farmer sold 3 bushels of apples last week. Then, she sold 6 bushels of apples this week.

Answers

Answer:

The answer is the first one, I put 5 pounds of birdseed and he ate 5 pounds, leaving you with none, or zero.

Step-by-step explanation:

Convert 10 pints to quarts.
Use the conversion rate 1 quart = 2 pints
Help

Answers

Answer: Your answer should be 5 quarts.

Step-by-step explanation:

After conversion 10 pints will be equal to 5 quarts .

Given,

Convert 10 pints to quarts.

Few unit conversions :

Conversion Factors:

1 mile = 5280 feet

1 foot = 12 inches

1 inch = 2.54 cm

1 cm = 10 mm

1 second = 1 s

Now,

Here 10 pints to quarts .

1 quart = 2 pints

Hence ,

10 quarts = 5 pints .

Know more about unit conversion,

https://brainly.com/question/14614674

#SPJ6

Use the image to answer the question below. 10 8 A B 10 8 10 -10 Which series of transformations would result in a figure that is congruent to the triangle ABC?​

Answers

Answer:

c b a

Step-by-step explanation:

Answer:

cba

Step-by-step explanation:

sorry if its wrong

Other Questions
prin ce insusiri se remarca sfantul si dreptul simeon Describe three situations in which rounding could come in handy. Which models show the problem 4/6 : 2/6 = 2? Check all that apply : PLEASE HELP ME I NEED THIS ASAP The number is odd. Its only two factors are prime numbers. The sum of the two factors is a factor of 36. The number is greater than 50 but less than 75. What is the mystery number? Locate the infinitive and determine how it is used.James hopes to succeed as a doctor. pls do this ill give brainliest!! Why would a plant cell not burst while an animal cell would undergo cytolysis in a hypotonic solution?The plant cell has a large central vacuole and creates a stronger exterior. The plant cell has a rigid cell wall and the pressure from the central vacuoles causes the stiffness of the plant and animal cells do not have a cell wall therefore the boundary of the cell is weaker. The plant cell is stronger because it has the chloroplast and causes a strong boundary that support the cell due to the glucose that is made from the process of photosynthesis. Why did Bolivar admire the American and French Revolutions? Which presidential candidate completely opposed the spread of slavery to western territories? What are the subjects of economics The Help When Minny calls Aibileen, what did Aibileen say about what Skeeter asked her? FIRST RIGHT GETS BRAINLIEST parker jogged 5 kilometers one mile is approximately equal to 2.2 kilometers which measurement is closest to the number of miles parker jogged. Is Jose Gutierrez de Lara a filibuster What was hayes's position on civil service reform? what did he do to promote it? Which quantity measures the rate at which a machine performs work? Which of the following statments does not represent the relationship between culture and one's moral reasoning?A.Cultures generally differ in what behaviors are considered appropriate.B.One's thought and beliefs are generally independent of cultural influences.C.Many cultures value an individual's rights and well-being when establishing moral standards.D.Many cultures consider situational influences when judging one's behavior as right or wrong. whats the answer?what is algebra WRITE IT OUT STEP BY STEP PLZ I NEED IT ASP1) Write a word problem using the information from the given equation and inequality. Be sure to include all seven parts and an appropriate question requesting a solution for the unknown (variable).The seven parts of a one-variable equation are as follows:1 correct variable term for the left side expression2 correct constant term for the left side expression3 correct operation between the terms of the left side expression4 correct equal sign or inequality symbol5 correct variable term for the right side expression6 correct constant term for the right side expression7 correct operation between the terms of the right side expressionEquation: -70.4x + 1,250 = -50.5x + 1,110 PLEASE HELP ME!! Its due in a couple of hours and Im having hard time understanding how to do these problems. Its 40 points, 10 for each problem.